This repository has been archived by the owner on Nov 29, 2022. It is now read-only.
-
Notifications
You must be signed in to change notification settings - Fork 22
/
Copy pathexample.tex
261 lines (242 loc) · 8.7 KB
/
example.tex
1
2
3
4
5
6
7
8
9
10
11
12
13
14
15
16
17
18
19
20
21
22
23
24
25
26
27
28
29
30
31
32
33
34
35
36
37
38
39
40
41
42
43
44
45
46
47
48
49
50
51
52
53
54
55
56
57
58
59
60
61
62
63
64
65
66
67
68
69
70
71
72
73
74
75
76
77
78
79
80
81
82
83
84
85
86
87
88
89
90
91
92
93
94
95
96
97
98
99
100
101
102
103
104
105
106
107
108
109
110
111
112
113
114
115
116
117
118
119
120
121
122
123
124
125
126
127
128
129
130
131
132
133
134
135
136
137
138
139
140
141
142
143
144
145
146
147
148
149
150
151
152
153
154
155
156
157
158
159
160
161
162
163
164
165
166
167
168
169
170
171
172
173
174
175
176
177
178
179
180
181
182
183
184
185
186
187
188
189
190
191
192
193
194
195
196
197
198
199
200
201
202
203
204
205
206
207
208
209
210
211
212
213
214
215
216
217
218
219
220
221
222
223
224
225
226
227
228
229
230
231
232
233
234
235
236
237
238
239
240
241
242
243
244
245
246
247
248
249
250
251
252
253
254
255
256
257
258
259
260
261
\documentclass{homework}
\name{Leonhard Euler}
\course{Math 101}
\term{Fall 1737}
\hwnum{1}
\begin{document}
\begin{problem}
\begin{parts}
\part
\label{1.a}
Prove that there is no rational number whose square is $2$.
\part
\label{1.b}
Let $p$ be a prime number and $n$ an integer greater than $1$.
Prove that there is no rational number whose $n$th power is $p$.
\end{parts}
\end{problem}
\begin{solution}
\ref{1.a}
Suppose that there is a rational number whose square is $2$.
Then we can find coprime nonzero integers $a$ and $b$ such that $a^2 / b^2 = 2$.
Then $a^2 = 2 b^2$, so $a^2$ is even.
It follows that $a$ is even, so $a = 2 k$ for some integer $k$.
Then $2 b^2 = 4 k^2$, so $b^2 = 2 k^2$, which implies that $b^2$ is even.
But then $b$ itself is even, which is a contradiction since $a$ is even and $a$
and $b$ are coprime.
Thus, there is no rational number whose square is $2$.
\ref{1.b}
If there were such a rational number, then it would be a root of the polynomial
$f = x^n - p$.
This polynomial is irreducible over $\mathbb{Z}$ by Eisenstein's criterion, and
so it is irreducible over $\mathbb{Q}$ by Gauss's Lemma.
In particular, $f$ has no roots in $\mathbb Q$, there is no rational number
whose $n$th power is $p$.
\end{solution}
\begin{problem}
Let $S$ be a dense subset of $\mathbb R$.
\begin{parts}[r]
\part
\label{2.i}
Suppose $f : \mathbb R \to \mathbb R$ is continuous and $f(x) = 0$ for all
$x \in S$.
\begin{parts}[A]
\part
\label{2.i.1}
Prove, using the \emph{definition} of denseness, that $f(x) = 0$ for all
$x \in \mathbb R$.
\part
\label{2.i.2}
Prove, using the fact that a subset of $\mathbb R$ is dense if and only if it is
\emph{sequentially dense}, that $f(x) = 0$ for all $x \in \mathbb R$.
\end{parts}
\part
\label{2.ii}
Suppose $f, g : \mathbb R \to \mathbb R$ are continuous and $f(x) = g(x)$ for
all $x \in S$.
Prove that $f(x) = g(x)$ for all $x \in \mathbb R$.
\end{parts}
\end{problem}
\begin{solution}
\ref{2.i}\ref{2.i.1}
Let $x \in \mathbb R$ be given, and pick any $\varepsilon > 0$.
Since $f$ is continuous at $x$, there exists a $\delta > 0$ such that
$|f(x) - f(y)| < \varepsilon$ for all $y \in \mathbb R$ such that
$|x - y| < \delta$.
Since $S$ is dense in $\mathbb R$, there exists a number
$y \in S \cap (x - \delta, x + \delta)$.
Then $f(y) = 0$ and $|x - y| < \delta$.
Therefore, $|f(x)| = |f(x) - f(y)| < \varepsilon$.
Since $|f(x)| < \varepsilon$ for an arbitrary $\varepsilon > 0$, it follows that
$f(x) = 0$.
Since $x$ was arbitrary, this shows that $f(x) = 0$ for all $x \in \mathbb R$.
\ref{2.i}\ref{2.i.2}
Since $S$ is a dense subset of $\mathbb R$, it is also sequentially dense (i.e.,
every real number is a limit of some sequence in $S$).
Let $x \in \mathbb R$ be given, and choose a sequence $\{x_n\}$ in $S$ that
converges to $x$.
For all $n \in \mathbb N$, we have $f(x_n) = 0$ since $x_n \in S$.
Thus, $\{f(x_n)\} \to 0$.
However, since $f$ is continuous and $\{x_n\} \to x$, it must be the case that
$\{f(x_n)\} \to f(x)$, so $f(x) = 0$.
Since $x$ was arbitrary, $f(x) = 0$ for all $x \in \mathbb R$.
\ref{2.ii}
Define $h : \mathbb R \to \mathbb R$ by $h(x) = f(x) - g(x)$.
Then $h$ is continuous and $h(x) = 0$ for all $x \in S$ (since $f(x) = g(x)$ for
all $x \in S$).
By part \ref{2.i}, it follows that $h(x) = 0$ for all $x \in \mathbb R$, so
therefore $f(x) = g(x)$ for all $x \in \mathbb R$.
\end{solution}
\begin{problem}[Ahlfors \S4.2.3 \#2, p. 123]
Prove that a function which is analytic in the whole plane and satisfies an
inequality $|f(z)| < |z|^n$ for some $n$ and all sufficiently large $|z|$
reduces to a polynomial.
\end{problem}
\begin{solution}
Since $f$ is entire, we may write $f$ as a power series centered at $0$ which
converges for every complex number $z$:
\begin{equation*}
f(z) = \sum_{k=0}^\infty a_k z^k.
\end{equation*}
Let $R > 0$ be large enough such that $|f(z)| < |z|^n$ whenever $|z| \geq R$.
Let $\gamma$ be the positively oriented circle of radius $R$ centered at the
origin, parametrized as $\gamma(t) = R e^{i t}$ for $0 \leq t \leq 2\pi$.
For each $k \geq 0$ we have
\begin{align*}
a_k
&= \frac{f^{(k)}(0)}{n!}
= \frac{1}{2 \pi i} \int_\gamma \frac{f(z)}{z^{k+1}} \, dz
= \frac{1}{2 \pi i} \int_0^{2\pi} \frac{f(\gamma(t))}{\gamma(t)^{k+1}} \gamma^\prime(t) \, dt
\\&= \frac{1}{2 \pi i} \int_0^{2\pi} \frac{f(R e^{i t})}{R^{k+1} e^{i(t+1) t}} i R e^{i t} \, dt
= \frac{1}{2 \pi R^k} \int_0^{2\pi} \frac{f(R e^{i t})}{e^{i k t}} \, dt,
\end{align*}
and hence
\begin{align*}
|a_k|
&= \frac{1}{2 \pi R^k} \left|\int_0^{2\pi} \frac{f(R e^{i t})}{e^{i k t}} \, dt\right|
\leq \frac{1}{2 \pi R^k} \int_0^{2\pi} \frac{|f(R e^{i t})|}{|e^{i k t}|} \, dt
\\&= \frac{1}{2 \pi R^k} \int_0^{2\pi} |f(R e^{i t})| \, dt
\leq \frac{1}{2 \pi R^k} \int_0^{2\pi} R^n \, dt
= \frac{R^n}{R^k}
= \frac{1}{R^{k - n}}.
\end{align*}
If $k > n$, then letting $R \to \infty$ gives $|a_k| = 0$.
Thus, we have $a_k = 0$ for $k > n$, so that
\begin{equation*}
f(z) = a_0 + a_1 x + \cdots + a_n x^n.
\end{equation*}
That is, $f$ is a polynomial.
\end{solution}
\begin{problem}
Prove that there are infinitely many prime numbers.
\end{problem}
\begin{solution}
Let $p_1,\ldots,p_n$ be a finite list of prime numbers.
The number $Q = p_1\cdots p_n + 1$ is coprime to $p_i$ for $i=1,\ldots,n$, so
$Q$ has a prime factor $q$ which is not among the primes $p_1, \ldots, p_n$.
Thus, given any finite list of primes numbers, we can find a prime number not in
the list.
It follows that there are infinitely many prime numbers.
\end{solution}
\begin{problem}
Let $p_n$ be the $n$th prime number.
Prove that the series
\begin{equation}
\label{eq:sum of reciprocals of primes}
\sum_{n=1}^\infty \frac{1}{p_n}
\end{equation}
diverges.
\end{problem}
\begin{solution}
Suppose, for the sake of contradiction, that
\eqref{eq:sum of reciprocals of primes} converges.
Then there is an integer $k$ such that
\begin{equation}
\label{eq:tail of sum of reciprocals of primes}
\sum_{n > k} \frac{1}{p_n} < \frac{1}{2}.
\end{equation}
Let $Q = p_1 \cdots p_k$, and, for every positive integer $N$, define
\begin{equation*}
S(N) = \sum_{n=1}^N \frac{1}{1 + n Q}.
\end{equation*}
Also, let $P(N)$ be the finite set of primes $q$ which divide $1 + n Q$ for at
least one $n \in \{1,\ldots,N\}$.
For each positive integer $n$, the integers $Q$ and $1 + n Q$ are coprime, so
none of the prime numbers $p_1,\ldots,p_k$ divide $1 + n Q$.
It follows that
\begin{equation}
\label{eq:P(N) subset}
P(N) \subseteq \{p_{k+1}, p_{k+2}, \ldots\}
\end{equation}
for every positive integer $N$.
If $m$ is another positive integer, then we define $A(N, m)$ to be the set of
all integers $n$ such that $1 \leq n \leq N$ and such that $1 + n Q$ has exactly
$m$ (not necessarily distinct) prime divisors.
Consider the sum
\begin{equation*}
S(N, m) = \sum_{n \in A(N, m)} \frac{1}{1 + n Q}.
\end{equation*}
Observe that $S(N, m) = 0$ for $m$ sufficiently large (since $A(N,m)$ is empty
for $m$ sufficiently large) and that
\begin{equation}
\label{eq:S(N) as sum}
S(N)
= \sum_{m=1}^\infty S(N, m).
\end{equation}
If $n \in A(N, m)$, then $1 + n Q = (q_1\cdots q_m)^{-1}$ for some
$q_1,\ldots,q_m \in P(N)$, so
\begin{equation*}
S(N, m)
\leq \sum_{q_1,\ldots,q_m \in P(N)} \frac{1}{q_1\cdots q_m}
= \left(\sum_{p \in P(N)} \frac{1}{p}\right)^{m}.
\end{equation*}
It then follows from \eqref{eq:tail of sum of reciprocals of primes} and
\eqref{eq:P(N) subset} that
\begin{equation*}
S(N, m)
\leq \left(\sum_{p \in P(N)} \frac{1}{p}\right)^{m}
\leq \left(\sum_{n > k} \frac{1}{p_n}\right)^{m}
< \frac{1}{2^m}
\end{equation*}
Now \eqref{eq:S(N) as sum} implies that
\begin{equation*}
S(N)
= \sum_{m=1}^\infty S(N, m)
\leq \sum_{m=1}^\infty \frac{1}{2^m} = 1.
\end{equation*}
Thus, the sequence $\{S(N)\}_{N=1}^\infty$ is a monotone increasing bounded
sequence, so it converges to the limit
\begin{equation*}
\lim_{N \to \infty} S(N)
= \lim_{N \to \infty} \sum_{n=1}^N \frac{1}{1 + n Q}
= \sum_{n=1}^\infty \frac{1}{1 + n Q}.
\end{equation*}
However, the series above diverges (e.g., by the integral test), so we have
reached a contradiction.
It follows that the series \eqref{eq:sum of reciprocals of primes} diverges.
\end{solution}
\begin{problem}[Atiyah-Macdonald 1.1]
Let $x$ be a nilpotent element of a ring $A$.
Show that $1 + x$ is a unit of $A$.
Deduce that the sum of a nilpotent element and a unit is a unit.
\end{problem}
\begin{solution}
Let $y = -x$.
Then $y$ is also nilpotent, so choose a positive integer $n$ such that
$y^n = 0$.
We have
\begin{align*}
(1 - y) (1 + y + y^2 + \cdots + y^{n-1})
= 1 - y^n
= 1
\end{align*}
which shows that $1 + x = 1 - y$ is a unit.
Next, let $x\in A$ be nilpotent and $u\in A$ a unit.
Then $u^{-1} x$ is nilpotent, so $1 + u^{-1}x$ is a unit by the first part.
Therefore
\begin{equation*}
u + x = u(1 + u^{-1} x).
\end{equation*}
Since it is a product of units, $u + x$ is a unit.
\end{solution}
\end{document}